You are on page 1of 3

Solution to the Warship Problem

I deeply encourage you to try this out yourself after going through my so-
lution, because in my opinion this problem test your overall understand of pro-
jectile motion very well, which is precisely the reason I want you to do this. It
is a bit difficult but doable, and if you can do it all by yourself, I think there
would hardly be any projectile problem that can trick you.

Following the arguement we made today, we expect there exist two special
angle L and H , bounded by which is the possible fire range of the enemys
cannon. Our first task is to determine these angles.

Let the intial velocity of the cannon ball be vi = 250 m


s , the height of the
peak be h = 1800 m , and the distance of the peak be D = 2500 m, we can first
write down the conditions for which the angles L and H occur:

D = vi cos t (1)
1
h = vi sin t gt2 (2)
2
Solving for t then plug into Eqn.(2):

D
t= (3)
vi cos
D 1 D
h = vi sin ( ) g( )2 (4)
vi cos 2 vi cos
1 D2 1
h = D tan g 2 (5)
2 vi cos2

1
Invoking the unfortunately unfamilar trig identity cos2 = (1 + tan2 ):

1 D2
h = D tan g 2 (1 + tan2 ) (6)
2 vi
2
1 D 1 D2
g 2 tan2 D tan + (h + g 2 ) = 0 (7)
2 vi 2 vi

1
Putting in the numbers, (I belive you forgot to divide by vi2 this morning
when we doing this problem)

1 D2
g = 490 (8)
2 vi2
D = 2500 (9)
2
1 D
h + g 2 = 2290 (10)
2 vi

Solving the resulting quadratic equation:

490 tan2 2500 tan + 2290 = 0 (11)


tan = 3.905 or 1.197 (12)

= 75.66 or 50.12 (13)

3
Obviously L = 50.12 which gives the furthest possible range and H =
75.66 which gives the closest possible range. Our next task is to determine the
exact value of these two range thereby finding the complete fire range of the
enemy ship.

We do that by following the usual way, let be the total travel time of
the cannon ball and notice that it take 12 for the ball to get to the top of its
trajectory where vy = 0:

1 vi sin
= (14)
2 g
2vi sin
x = vi cos = vi cos ( ) (15)
g

Invoking another trig identity (hopefully you remmember this one, this is a
particular useful one to remmember) sin 2x = 2 sin x cos x:

vi2 sin (2)


x= (16)
g

(Eqn.16 is also a very useful result to remmember for projectile problem)

2
4

Finally, pluging in the numbers to Eqn., we have (rounded to the significant


figures)

vi2 sin (2L )


xL = = 6280 m (17)
g
v 2 sin (2H )
xH = i = 3060 m (18)
g

But since these distance is measured relative to the emeny ship, we need to
subtract from these distance (2500 m + 300 m) = 2800 m to get the distance
from the shore.

The final result: the possible enemy fire range is between 260 m to 3480 m
from the shore, hence our ship is safe if we are at x < 260 m or x > 3480 m
from the shore.

You might also like